K
Khách

Hãy nhập câu hỏi của bạn vào đây, nếu là tài khoản VIP, bạn sẽ được ưu tiên trả lời.

28 tháng 8 2016

Ta có :

\(31\left(xyzt+xy+xt+zt+1\right)=40\left(yzt+y+t\right)\)

\(\Rightarrow\frac{xyzt+xy+xt+zt+1}{yzt+y+t}=\frac{40}{31}\)

\(\Rightarrow\frac{x\left(yzt+y+t\right)+zt+1}{yzt+y+t}=\frac{40}{31}\)

\(\Rightarrow x+\frac{zt+1}{yzt+y+t}=\frac{40}{31}\)

\(\Rightarrow x+\frac{1}{\left(\frac{yzt+y+t}{zt+1}\right)}=\frac{40}{31}\)

\(\Rightarrow x+\frac{1}{\left(y+\frac{t}{zt+1}\right)}=\frac{40}{31}\)

\(\Rightarrow x+\frac{1}{y+\frac{1}{\left(\frac{zt+1}{t}\right)}}=\frac{40}{31}\)

\(\Rightarrow x+\frac{1}{y+\frac{1}{z+\frac{1}{t}}}=\frac{40}{31}\)

\(\frac{40}{31}< \frac{62}{31}=2\Rightarrow x< 2\)

Với x = 0; có :

\(\frac{1}{y+\frac{1}{z+\frac{1}{t}}}=\frac{40}{31}\)

\(\Rightarrow y+\frac{1}{z+\frac{1}{t}}=\frac{31}{40}\)

Mà \(\frac{31}{40}< 1\Rightarrow y< 1\Rightarrow y=0\)

\(\Rightarrow\frac{1}{z+\frac{1}{t}}=\frac{31}{40}\)

\(\Rightarrow z+\frac{1}{t}=\frac{40}{31}\)

\(\cdot z=0\Rightarrow t=\frac{31}{40}\notin Z\)(Loại )

\(\cdot z=1\Rightarrow t=\frac{31}{9}\notin Z\)(Loại )

Với \(x=1;\)ta có :

\(\frac{1}{y+\frac{1}{z+\frac{1}{t}}}=\frac{40}{31}-1\)

\(\Rightarrow\frac{1}{y+\frac{1}{z+\frac{1}{t}}}=\frac{9}{31}\)

\(\Rightarrow y+\frac{1}{z+\frac{1}{t}}=\frac{31}{9}\)

\(\frac{31}{9}< \frac{36}{9}=4\Rightarrow y< 4\)

\(\cdot y=0\Rightarrow z+\frac{1}{t}=\frac{9}{31}\Rightarrow z=0\Rightarrow t=\frac{31}{9}\notin Z\)(Loại)

\(\cdot y=1\Rightarrow z+\frac{1}{t}=\frac{9}{22}\Rightarrow z=0\Rightarrow t=\frac{22}{9}\notin Z\)(Loại)

\(\cdot y=2\Rightarrow z+\frac{1}{t}=\frac{9}{13}\Rightarrow z=0\Rightarrow t=\frac{13}{9}\notin Z\)(Loại )

\(\cdot y=3\Rightarrow z+\frac{1}{t}=\frac{9}{4}\)

\(\frac{9}{4}< 3\Rightarrow z< 3\)

  1. \(z=0\Rightarrow t=\frac{4}{9}\notin Z\)
  2. \(z=1\Rightarrow t=\frac{4}{5}\notin Z\)
  3. \(z=2\Rightarrow t=4\)( Thỏa mãn )

Vậy \(x=1;y=3;z=2;t=4.\)

AH
Akai Haruma
Giáo viên
24 tháng 2 2020

Lời giải:

Đặt biểu thức vế trái là $A$

Áp dụng BĐT Bunhiacopxky:

\(A[x(yz+zt+ty)+y(xz+zt+xt)+z(xt+yt+xy)+t(xy+yz+xz)]\geq \left(\frac{1}{x}+\frac{1}{y}+\frac{1}{z}+\frac{1}{t}\right)^2\)
Vì $xyzt=1$ nên:

\(x(yz+zt+ty)+y(xz+zt+xt)+z(xt+yt+xy)+t(xy+yz+xz)=\frac{1}{t}+\frac{1}{y}+\frac{1}{z}+\frac{1}{t}+\frac{1}{x}+\frac{1}{z}+\frac{1}{y}+\frac{1}{x}+\frac{1}{t}+\frac{1}{z}+\frac{1}{x}+\frac{1}{y}=3\left(\frac{1}{x}+\frac{1}{y}+\frac{1}{z}+\frac{1}{t}\right)\)

Do đó:

$A. 3\left(\frac{1}{x}+\frac{1}{y}+\frac{1}{z}+\frac{1}{t}\right)\geq \left(\frac{1}{x}+\frac{1}{y}+\frac{1}{z}+\frac{1}{t}\right)^2$

$\Rightarrow A\geq \frac{\frac{1}{x}+\frac{1}{y}+\frac{1}{z}+\frac{1}{t}}{3}$

Áp dụng BĐT AM-GM: \frac{1}{x}+\frac{1}{y}+\frac{1}{z}+\frac{1}{t}\geq 4\sqrt[4]{\frac{1}{xyzt}}=4$

Vậy $A\geq \frac{4}{3}$ (đpcm)

24 tháng 2 2020

x,y,z,t có dương không mà dùng AM-GM (hay đề sai)

1 tháng 4 2018

ai  mà biết hả

1 tháng 4 2018

Bạn vô duyên quá đấy!